If $1 > a-b > 0$ then $[a]geq[b]$If $a$ and $b$ are positive real numbers, then $a + b geq 2 sqrtab$.Proof if $n_k < n_k+1$ for all $k in mathbbN$, then $n_k geq k$ for all $k in mathbbN$.If $f(n) geq g(n)$ for all $n geq n_0$ then $f(n) geq cg(n)$ for all $n geq n_1$Inequality $2a^nb^nc^n+1geq a^2n+b^2n+c^2n$Suppose $0<a<b$. Prove for all $ngeq 2$, $0< sqrt[n]a< sqrt[n]b$.On some iterated inequalities and $x geq 5$Show that $2^n geq (n +2)^2$ for all $n geq 6$Induction for $3^ngeq n^2$What about $f$ if $fracdleft(xf(x)-xright)dx geq 0$Prove or disprove $|a| geq 2|b| implies |a+b| geq |b|$

How difficult is it to simply disable/disengage the MCAS on Boeing 737 Max 8 & 9 Aircraft?

What options are left, if Britain cannot decide?

What is the significance behind "40 days" that often appears in the Bible?

Gantt Chart like rectangles with log scale

Why do Australian milk farmers need to protest supermarkets' milk price?

Why would a flight no longer considered airworthy be redirected like this?

Why one should not leave fingerprints on bulbs and plugs?

How can I track script which gives me "command not found" right after the login?

Is it possible to upcast ritual spells?

If curse and magic is two sides of the same coin, why the former is forbidden?

What exactly is this small puffer fish doing and how did it manage to accomplish such a feat?

Gravity magic - How does it work?

How to write cleanly even if my character uses expletive language?

What has been your most complicated TikZ drawing?

Define, (actually define) the "stability" and "energy" of a compound

Unexpected result from ArcLength

Is a party consisting of only a bard, a cleric, and a warlock functional long-term?

Bach's Toccata and Fugue in D minor breaks the "no parallel octaves" rule?

Error in Twin Prime Conjecture

Dice rolling probability game

Continuity of Linear Operator Between Hilbert Spaces

Provisioning profile doesn't include the application-identifier and keychain-access-groups entitlements

How do I hide Chekhov's Gun?

Credit cards used everywhere in Singapore or Malaysia?



If $1 > a-b > 0$ then $[a]geq[b]$


If $a$ and $b$ are positive real numbers, then $a + b geq 2 sqrtab$.Proof if $n_k < n_k+1$ for all $k in mathbbN$, then $n_k geq k$ for all $k in mathbbN$.If $f(n) geq g(n)$ for all $n geq n_0$ then $f(n) geq cg(n)$ for all $n geq n_1$Inequality $2a^nb^nc^n+1geq a^2n+b^2n+c^2n$Suppose $0<a<b$. Prove for all $ngeq 2$, $0< sqrt[n]a< sqrt[n]b$.On some iterated inequalities and $x geq 5$Show that $2^n geq (n +2)^2$ for all $n geq 6$Induction for $3^ngeq n^2$What about $f$ if $fracdleft(xf(x)-xright)dx geq 0$Prove or disprove $|a| geq 2|b| implies |a+b| geq |b|$













-1












$begingroup$


what I can do for The case $[a]<[b]$.



I know that $[a]>[b]Rightarrow [a]geq [b]+1$



EDIT:



I have realized that the Initial assumption $[a]=[b]$ is false therefore I have changed it to $[a]geq [b]$










share|cite|improve this question











$endgroup$











  • $begingroup$
    For clarity, what does $[x]$ mean in this context? I've usually seen this to refer to the rounding function, i.e. rounding $x$ to the nearest integer, but a few counterexamples immediately come to mind in that instance
    $endgroup$
    – Eevee Trainer
    Mar 11 at 21:02










  • $begingroup$
    $[x]$ is the biggest integer for which we have $leq x$. Can you give me an counterexample?
    $endgroup$
    – New2Math
    Mar 11 at 21:04






  • 3




    $begingroup$
    What if $a=1.1$ and $b=0.9$?
    $endgroup$
    – clark
    Mar 11 at 21:06










  • $begingroup$
    Okay so you're referring to the floor function then @New2Math - so my counterexample would be irrelevant. (Were it rounding and not floor, I was thinking $a=0.6,b=0.4$.) clark's counterexample works for the floor function though.
    $endgroup$
    – Eevee Trainer
    Mar 11 at 21:10










  • $begingroup$
    @clark I see now that this assumption is false but the assumption $a-b>1Rightarrow [a]>[b]$ is true right, also If I would have changed the assumption in the question to $geq$ then it would be right, right?
    $endgroup$
    – New2Math
    Mar 11 at 21:10
















-1












$begingroup$


what I can do for The case $[a]<[b]$.



I know that $[a]>[b]Rightarrow [a]geq [b]+1$



EDIT:



I have realized that the Initial assumption $[a]=[b]$ is false therefore I have changed it to $[a]geq [b]$










share|cite|improve this question











$endgroup$











  • $begingroup$
    For clarity, what does $[x]$ mean in this context? I've usually seen this to refer to the rounding function, i.e. rounding $x$ to the nearest integer, but a few counterexamples immediately come to mind in that instance
    $endgroup$
    – Eevee Trainer
    Mar 11 at 21:02










  • $begingroup$
    $[x]$ is the biggest integer for which we have $leq x$. Can you give me an counterexample?
    $endgroup$
    – New2Math
    Mar 11 at 21:04






  • 3




    $begingroup$
    What if $a=1.1$ and $b=0.9$?
    $endgroup$
    – clark
    Mar 11 at 21:06










  • $begingroup$
    Okay so you're referring to the floor function then @New2Math - so my counterexample would be irrelevant. (Were it rounding and not floor, I was thinking $a=0.6,b=0.4$.) clark's counterexample works for the floor function though.
    $endgroup$
    – Eevee Trainer
    Mar 11 at 21:10










  • $begingroup$
    @clark I see now that this assumption is false but the assumption $a-b>1Rightarrow [a]>[b]$ is true right, also If I would have changed the assumption in the question to $geq$ then it would be right, right?
    $endgroup$
    – New2Math
    Mar 11 at 21:10














-1












-1








-1


1



$begingroup$


what I can do for The case $[a]<[b]$.



I know that $[a]>[b]Rightarrow [a]geq [b]+1$



EDIT:



I have realized that the Initial assumption $[a]=[b]$ is false therefore I have changed it to $[a]geq [b]$










share|cite|improve this question











$endgroup$




what I can do for The case $[a]<[b]$.



I know that $[a]>[b]Rightarrow [a]geq [b]+1$



EDIT:



I have realized that the Initial assumption $[a]=[b]$ is false therefore I have changed it to $[a]geq [b]$







inequality






share|cite|improve this question















share|cite|improve this question













share|cite|improve this question




share|cite|improve this question








edited Mar 11 at 21:16







New2Math

















asked Mar 11 at 20:59









New2MathNew2Math

11812




11812











  • $begingroup$
    For clarity, what does $[x]$ mean in this context? I've usually seen this to refer to the rounding function, i.e. rounding $x$ to the nearest integer, but a few counterexamples immediately come to mind in that instance
    $endgroup$
    – Eevee Trainer
    Mar 11 at 21:02










  • $begingroup$
    $[x]$ is the biggest integer for which we have $leq x$. Can you give me an counterexample?
    $endgroup$
    – New2Math
    Mar 11 at 21:04






  • 3




    $begingroup$
    What if $a=1.1$ and $b=0.9$?
    $endgroup$
    – clark
    Mar 11 at 21:06










  • $begingroup$
    Okay so you're referring to the floor function then @New2Math - so my counterexample would be irrelevant. (Were it rounding and not floor, I was thinking $a=0.6,b=0.4$.) clark's counterexample works for the floor function though.
    $endgroup$
    – Eevee Trainer
    Mar 11 at 21:10










  • $begingroup$
    @clark I see now that this assumption is false but the assumption $a-b>1Rightarrow [a]>[b]$ is true right, also If I would have changed the assumption in the question to $geq$ then it would be right, right?
    $endgroup$
    – New2Math
    Mar 11 at 21:10

















  • $begingroup$
    For clarity, what does $[x]$ mean in this context? I've usually seen this to refer to the rounding function, i.e. rounding $x$ to the nearest integer, but a few counterexamples immediately come to mind in that instance
    $endgroup$
    – Eevee Trainer
    Mar 11 at 21:02










  • $begingroup$
    $[x]$ is the biggest integer for which we have $leq x$. Can you give me an counterexample?
    $endgroup$
    – New2Math
    Mar 11 at 21:04






  • 3




    $begingroup$
    What if $a=1.1$ and $b=0.9$?
    $endgroup$
    – clark
    Mar 11 at 21:06










  • $begingroup$
    Okay so you're referring to the floor function then @New2Math - so my counterexample would be irrelevant. (Were it rounding and not floor, I was thinking $a=0.6,b=0.4$.) clark's counterexample works for the floor function though.
    $endgroup$
    – Eevee Trainer
    Mar 11 at 21:10










  • $begingroup$
    @clark I see now that this assumption is false but the assumption $a-b>1Rightarrow [a]>[b]$ is true right, also If I would have changed the assumption in the question to $geq$ then it would be right, right?
    $endgroup$
    – New2Math
    Mar 11 at 21:10
















$begingroup$
For clarity, what does $[x]$ mean in this context? I've usually seen this to refer to the rounding function, i.e. rounding $x$ to the nearest integer, but a few counterexamples immediately come to mind in that instance
$endgroup$
– Eevee Trainer
Mar 11 at 21:02




$begingroup$
For clarity, what does $[x]$ mean in this context? I've usually seen this to refer to the rounding function, i.e. rounding $x$ to the nearest integer, but a few counterexamples immediately come to mind in that instance
$endgroup$
– Eevee Trainer
Mar 11 at 21:02












$begingroup$
$[x]$ is the biggest integer for which we have $leq x$. Can you give me an counterexample?
$endgroup$
– New2Math
Mar 11 at 21:04




$begingroup$
$[x]$ is the biggest integer for which we have $leq x$. Can you give me an counterexample?
$endgroup$
– New2Math
Mar 11 at 21:04




3




3




$begingroup$
What if $a=1.1$ and $b=0.9$?
$endgroup$
– clark
Mar 11 at 21:06




$begingroup$
What if $a=1.1$ and $b=0.9$?
$endgroup$
– clark
Mar 11 at 21:06












$begingroup$
Okay so you're referring to the floor function then @New2Math - so my counterexample would be irrelevant. (Were it rounding and not floor, I was thinking $a=0.6,b=0.4$.) clark's counterexample works for the floor function though.
$endgroup$
– Eevee Trainer
Mar 11 at 21:10




$begingroup$
Okay so you're referring to the floor function then @New2Math - so my counterexample would be irrelevant. (Were it rounding and not floor, I was thinking $a=0.6,b=0.4$.) clark's counterexample works for the floor function though.
$endgroup$
– Eevee Trainer
Mar 11 at 21:10












$begingroup$
@clark I see now that this assumption is false but the assumption $a-b>1Rightarrow [a]>[b]$ is true right, also If I would have changed the assumption in the question to $geq$ then it would be right, right?
$endgroup$
– New2Math
Mar 11 at 21:10





$begingroup$
@clark I see now that this assumption is false but the assumption $a-b>1Rightarrow [a]>[b]$ is true right, also If I would have changed the assumption in the question to $geq$ then it would be right, right?
$endgroup$
– New2Math
Mar 11 at 21:10











1 Answer
1






active

oldest

votes


















0












$begingroup$

If $a geq b$ then it immediately follows that $lfloor a rfloor geq lfloor brfloor$. So we don't even need the upper bound.



(All this says is that the floor function is monotone increasing.)






share|cite|improve this answer









$endgroup$












    Your Answer





    StackExchange.ifUsing("editor", function ()
    return StackExchange.using("mathjaxEditing", function ()
    StackExchange.MarkdownEditor.creationCallbacks.add(function (editor, postfix)
    StackExchange.mathjaxEditing.prepareWmdForMathJax(editor, postfix, [["$", "$"], ["\\(","\\)"]]);
    );
    );
    , "mathjax-editing");

    StackExchange.ready(function()
    var channelOptions =
    tags: "".split(" "),
    id: "69"
    ;
    initTagRenderer("".split(" "), "".split(" "), channelOptions);

    StackExchange.using("externalEditor", function()
    // Have to fire editor after snippets, if snippets enabled
    if (StackExchange.settings.snippets.snippetsEnabled)
    StackExchange.using("snippets", function()
    createEditor();
    );

    else
    createEditor();

    );

    function createEditor()
    StackExchange.prepareEditor(
    heartbeatType: 'answer',
    autoActivateHeartbeat: false,
    convertImagesToLinks: true,
    noModals: true,
    showLowRepImageUploadWarning: true,
    reputationToPostImages: 10,
    bindNavPrevention: true,
    postfix: "",
    imageUploader:
    brandingHtml: "Powered by u003ca class="icon-imgur-white" href="https://imgur.com/"u003eu003c/au003e",
    contentPolicyHtml: "User contributions licensed under u003ca href="https://creativecommons.org/licenses/by-sa/3.0/"u003ecc by-sa 3.0 with attribution requiredu003c/au003e u003ca href="https://stackoverflow.com/legal/content-policy"u003e(content policy)u003c/au003e",
    allowUrls: true
    ,
    noCode: true, onDemand: true,
    discardSelector: ".discard-answer"
    ,immediatelyShowMarkdownHelp:true
    );



    );













    draft saved

    draft discarded


















    StackExchange.ready(
    function ()
    StackExchange.openid.initPostLogin('.new-post-login', 'https%3a%2f%2fmath.stackexchange.com%2fquestions%2f3144238%2fif-1-a-b-0-then-a-geqb%23new-answer', 'question_page');

    );

    Post as a guest















    Required, but never shown

























    1 Answer
    1






    active

    oldest

    votes








    1 Answer
    1






    active

    oldest

    votes









    active

    oldest

    votes






    active

    oldest

    votes









    0












    $begingroup$

    If $a geq b$ then it immediately follows that $lfloor a rfloor geq lfloor brfloor$. So we don't even need the upper bound.



    (All this says is that the floor function is monotone increasing.)






    share|cite|improve this answer









    $endgroup$

















      0












      $begingroup$

      If $a geq b$ then it immediately follows that $lfloor a rfloor geq lfloor brfloor$. So we don't even need the upper bound.



      (All this says is that the floor function is monotone increasing.)






      share|cite|improve this answer









      $endgroup$















        0












        0








        0





        $begingroup$

        If $a geq b$ then it immediately follows that $lfloor a rfloor geq lfloor brfloor$. So we don't even need the upper bound.



        (All this says is that the floor function is monotone increasing.)






        share|cite|improve this answer









        $endgroup$



        If $a geq b$ then it immediately follows that $lfloor a rfloor geq lfloor brfloor$. So we don't even need the upper bound.



        (All this says is that the floor function is monotone increasing.)







        share|cite|improve this answer












        share|cite|improve this answer



        share|cite|improve this answer










        answered Mar 11 at 21:27









        Daniel McLauryDaniel McLaury

        15.9k32981




        15.9k32981



























            draft saved

            draft discarded
















































            Thanks for contributing an answer to Mathematics Stack Exchange!


            • Please be sure to answer the question. Provide details and share your research!

            But avoid


            • Asking for help, clarification, or responding to other answers.

            • Making statements based on opinion; back them up with references or personal experience.

            Use MathJax to format equations. MathJax reference.


            To learn more, see our tips on writing great answers.




            draft saved


            draft discarded














            StackExchange.ready(
            function ()
            StackExchange.openid.initPostLogin('.new-post-login', 'https%3a%2f%2fmath.stackexchange.com%2fquestions%2f3144238%2fif-1-a-b-0-then-a-geqb%23new-answer', 'question_page');

            );

            Post as a guest















            Required, but never shown





















































            Required, but never shown














            Required, but never shown












            Required, but never shown







            Required, but never shown

































            Required, but never shown














            Required, but never shown












            Required, but never shown







            Required, but never shown







            Popular posts from this blog

            Lowndes Grove History Architecture References Navigation menu32°48′6″N 79°57′58″W / 32.80167°N 79.96611°W / 32.80167; -79.9661132°48′6″N 79°57′58″W / 32.80167°N 79.96611°W / 32.80167; -79.9661178002500"National Register Information System"Historic houses of South Carolina"Lowndes Grove""+32° 48' 6.00", −79° 57' 58.00""Lowndes Grove, Charleston County (260 St. Margaret St., Charleston)""Lowndes Grove"The Charleston ExpositionIt Happened in South Carolina"Lowndes Grove (House), Saint Margaret Street & Sixth Avenue, Charleston, Charleston County, SC(Photographs)"Plantations of the Carolina Low Countrye

            random experiment with two different functions on unit interval Announcing the arrival of Valued Associate #679: Cesar Manara Planned maintenance scheduled April 23, 2019 at 00:00UTC (8:00pm US/Eastern)Random variable and probability space notionsRandom Walk with EdgesFinding functions where the increase over a random interval is Poisson distributedNumber of days until dayCan an observed event in fact be of zero probability?Unit random processmodels of coins and uniform distributionHow to get the number of successes given $n$ trials , probability $P$ and a random variable $X$Absorbing Markov chain in a computer. Is “almost every” turned into always convergence in computer executions?Stopped random walk is not uniformly integrable

            How should I support this large drywall patch? Planned maintenance scheduled April 23, 2019 at 00:00UTC (8:00pm US/Eastern) Announcing the arrival of Valued Associate #679: Cesar Manara Unicorn Meta Zoo #1: Why another podcast?How do I cover large gaps in drywall?How do I keep drywall around a patch from crumbling?Can I glue a second layer of drywall?How to patch long strip on drywall?Large drywall patch: how to avoid bulging seams?Drywall Mesh Patch vs. Bulge? To remove or not to remove?How to fix this drywall job?Prep drywall before backsplashWhat's the best way to fix this horrible drywall patch job?Drywall patching using 3M Patch Plus Primer